Difference between revisions of "2004 AMC 8 Problems/Problem 14"
Pimaster314 (talk | contribs) m (→Solution 2) |
m (→Solution 2) |
||
Line 25: | Line 25: | ||
==Solution 2== | ==Solution 2== | ||
− | Apply [[Pick's Theorem]] on the figure, and you will get <cmath>\boxed{\textbf{(C)}\ 22\frac12}</cmath> | + | Apply [[Pick's Theorem]] on the figure, and you will get 5/2+21-1 which = <cmath>\boxed{\textbf{(C)}\ 22\frac12}</cmath> |
==See Also== | ==See Also== | ||
{{AMC8 box|year=2004|num-b=13|num-a=15}} | {{AMC8 box|year=2004|num-b=13|num-a=15}} | ||
{{MAA Notice}} | {{MAA Notice}} |
Revision as of 14:01, 2 April 2021
Contents
Problem
What is the area enclosed by the geoboard quadrilateral below?
Solution
Assign points to each of the four vertices and use the shoelace theorem to find the area. Letting the bottom left corner be , counting the boxes, the points would be and . Applying the Shoelace Theorem,
Solution 2
Apply Pick's Theorem on the figure, and you will get 5/2+21-1 which =
See Also
2004 AMC 8 (Problems • Answer Key • Resources) | ||
Preceded by Problem 13 |
Followed by Problem 15 | |
1 • 2 • 3 • 4 • 5 • 6 • 7 • 8 • 9 • 10 • 11 • 12 • 13 • 14 • 15 • 16 • 17 • 18 • 19 • 20 • 21 • 22 • 23 • 24 • 25 | ||
All AJHSME/AMC 8 Problems and Solutions |
The problems on this page are copyrighted by the Mathematical Association of America's American Mathematics Competitions.